Đến nội dung

Hình ảnh

$\frac{a^2}{b}+\frac{b^2}{c}+\frac{c^2}{a}+4(ab+bc+ac)\geq 15$

- - - - -

  • Please log in to reply
Chủ đề này có 6 trả lời

#1
binhnhaukhong

binhnhaukhong

    Sĩ quan

  • Thành viên
  • 343 Bài viết

Bài 1: Cho $x,y,z$ không âm sao cho $xy+yz+xz>0$.Chứng minh rằng:

 

$(xy+yz+xz)\left [ \frac{1}{(x+y)^2}+\frac{1}{(y+z)^2}+\frac{1}{(x+z)^2} \right ]\geq \frac{9}{4}+\frac{xyz(x^3+y^3+z^3-3xyz)}{(x+y)^2(y+z)^2(z+x)^2}$

 

Bài 2: Cho $a,b,c>0$ thỏa $a+b+c=3$.Chứng minh rằng:

 

$\frac{a^2}{b}+\frac{b^2}{c}+\frac{c^2}{a}+4(ab+bc+ac)\geq 15$

 

Bài 3: Cho $a,b,c$ không âm thỏa $a+b+c=3$.Chứng minh rằng:

 

$a^3b+b^3c+c^3a+\frac{1419abc}{256}\leq \frac{2187}{256}$

 

Bài 4: Cho $a,b,c$ thực dương.Chứng minh rằng:

 

$\frac{a^3+b^3+c^3}{8abc}+\frac{a(b+c)}{b^2+c^2}+\frac{b(a+c)}{a^2+c^2}+\frac{c(a+b)}{a^2+b^2}\geq \frac{27}{8}$

 

Bài 5: Cho $a,b,c>0, a^2+b^2+c^2=3$.Chứng minh rằng:

 

$\frac{a^2}{b}+\frac{b^2}{c}+\frac{c^2}{a}+2abc\geq 5$

 

Bài 6: Cho $a,b,c$ thực dương.Chứng minh rằng:

 

$\sum \frac{a^2}{b^2+bc+c^2}\geq \frac{(a^2+b^2+c^2)(a^3+b^3+c^3+3abc)}{(ab+bc+ac)\left [ \sum ab(a+b) \right ]}$

 

Bài 7: Cho $a,b,c>0$ thỏa $a+b+c=1$.Chứng minh rằng:

 

$\sqrt{a^2+abc}+\sqrt{b^2+abc}+\sqrt{c^2+abc}\leq \frac{2}{\sqrt{3}}$

 

Bài 8: $a,b,c\geq 0, a^2+b^2+c^2=4$:

 

$a^3b+b^3c+c^3a\leq \frac{256}{75}+\frac{16}{15}(ab+bc+ac)$

 

Bài 9: $a,b,c\geq 0,a^2+b^2+c^2=1$

 

$\frac{1-ab}{7-3ac}+\frac{1-bc}{7-3ab}+\frac{1-ca}{7-3bc}\geq \frac{1}{3}$

 

Bài 10: $a,b,c$ là các số thực khác 0 .$a^2+b^2+c^2=1$.Chứng minh:

 

$\frac{1-3ab}{1-2ac}+\frac{1-3bc}{1-2ab}+\frac{1-3ca}{1-2bc}\geq 0$

 

Spoiler


Bài viết đã được chỉnh sửa nội dung bởi binhnhaukhong: 12-07-2015 - 15:28

Quy Ẩn Giang Hồ. 

So goodbye!

 

:off:  :off:  :off:  :off:  :off:  :off: 


#2
Senju Hashirama

Senju Hashirama

    Hạ sĩ

  • Thành viên
  • 69 Bài viết

Bài 1: Cho $x,y,z$ không âm sao cho $xy+yz+xz>0$.Chứng minh rằng:

 

$(xy+yz+xz)\left [ \frac{1}{(x+y)^2}+\frac{1}{(y+z)^2}+\frac{1}{(x+z)^2} \right ]\geq \frac{9}{4}+\frac{xyz(x^3+y^3+z^3-3xyz)}{(x+y)^2(y+z)^2(z+x)^2}$

 

Bài 2: Cho $a,b,c>0$ thỏa $a+b+c=3$.Chứng minh rằng:

 

$\frac{a^2}{b}+\frac{b^2}{c}+\frac{c^2}{a}+4(ab+bc+ac)\geq 15$

 

Bài 3: Cho $a,b,c$ không âm thỏa $a+b+c=3$.Chứng minh rằng:

 

$a^3b+b^3c+c^3a+\frac{1419abc}{256}\leq \frac{2187}{256}$

 

Bài 4: Cho $a,b,c$ thực dương.Chứng minh rằng:

 

$\frac{a^3+b^3+c^3}{8abc}+\frac{a(b+c)}{b^2+c^2}+\frac{b(a+c)}{a^2+c^2}+\frac{c(a+b)}{a^2+b^2}\geq \frac{27}{8}$

 

Bài 5: Cho $a,b,c>0, a^2+b^2+c^2=3$.Chứng minh rằng:

 

$\frac{a^2}{b}+\frac{b^2}{c}+\frac{c^2}{a}+2abc\geq 5$

 

Bài 6: Cho $a,b,c$ thực dương.Chứng minh rằng:

 

$\sum \frac{a^2}{b^2+bc+c^2}\geq \frac{(a^2+b^2+c^2)(a^3+b^3+c^3+3abc)}{(ab+bc+ac)\left [ \sum ab(a+b) \right ]}$

 

Bài 7: Cho $a,b,c>0$ thỏa $a+b+c=1$.Chứng minh rằng:

 

$\sqrt{a^2+abc}+\sqrt{b^2+abc}+\sqrt{c^2+abc}\leq \frac{2}{\sqrt{3}}$

 

Bài 8: $a,b,c\geq 0, a^2+b^2+c^2=4$:

 

$a^3b+b^3c+c^3a\leq \frac{256}{75}+\frac{16}{15}(ab+bc+ac)$

 

Bài 9: $a,b,c\geq 0,a^2+b^2+c^2=1$

 

$\frac{1-ab}{7-3ac}+\frac{1-bc}{7-3ab}+\frac{1-ca}{7-3bc}\geq \frac{1}{3}$

 

Bài 10: $a,b,c$ là các số thực khác 0 .$a^2+b^2+c^2=1$.Chứng minh:

 

$\frac{1-3ab}{1-2ac}+\frac{1-3bc}{1-2ab}+\frac{1-3ca}{1-2bc}\geq 0$

 

Spoiler

Bài 2 :

                             Ta có Bđt  $\Leftrightarrow \sum \frac{a^{2}}{b}\geq 15-4\sum ab\geq 15-4.\frac{\left ( \sum a \right )^{2}}{3}=3$

 

                                       

Ta sẽ chứng minh :  

                                   $\sum \frac{a^{2}}{b}\geq \frac{3\sum a^{2}}{\sum a}$

                        

Thật vậy , BĐT tương đương :  

                                                    $\sum \frac{a^{2}}{b}-\sum a\geq \frac{\sum a^{2}}{\sum a}-\sum a$

 

                                             $\Leftrightarrow \sum \frac{(a-b)^{2}}{b}\geq \frac{\sum (a-b)^{2}}{\sum a}$

 

                                  $\Leftrightarrow S_{a}\left ( b-c \right )^{2}+S_{b}\left ( c-a \right )^{2}+S_{c}\left ( a-b \right )^{2}\geq 0$ (luôn đúng ) 

   

      Tại :

                                       $S_{a}=\frac{1}{c}-\frac{1}{a+b+c}=\frac{a+b}{c\left ( a+b+c \right )}>0$ , tương tự với $S_{b},S_{c}>0

              Dễ thấy :

 

                                                $\sum a^{2}\geq \frac{(\sum a)^{2}}{3}=\sum a\Rightarrow \frac{3\sum a^{2}}{\sum a}\geq 3$

 

                      Từ đây ta có điều phải chứng minh                                                                


Bài viết đã được chỉnh sửa nội dung bởi Senju Hashirama: 12-07-2015 - 13:53


#3
Senju Hashirama

Senju Hashirama

    Hạ sĩ

  • Thành viên
  • 69 Bài viết

Bài 5 : tương tự Bài 2 

                              Chỉ cần để ý đánh giá :  $\sum a^{2}\geq 3\sqrt[3]{a^{2}b^{2}c^{2}}\Rightarrow abc\leq 1\Rightarrow 5-2abc\geq 3$


Bài viết đã được chỉnh sửa nội dung bởi Senju Hashirama: 12-07-2015 - 13:59


#4
Hoang Nhat Tuan

Hoang Nhat Tuan

    Hỏa Long

  • Thành viên
  • 974 Bài viết

Bài 2 :

                             Ta có Bđt  $\Leftrightarrow \sum \frac{a^{2}}{b}\geq 15-4\sum ab\geq 15-4.\frac{\left ( \sum a \right )^{2}}{3}=3$

 

                                       

Ta sẽ chứng minh :  

                                   $\sum \frac{a^{2}}{b}\geq \frac{3\sum a^{2}}{\sum a}$

                        

Thật vậy , BĐT tương đương :  

                                                    $\sum \frac{a^{2}}{b}-\sum a\geq \frac{\sum a^{2}}{\sum a}-\sum a$

 

                                             $\Leftrightarrow \sum \frac{(a-b)^{2}}{b}\geq \frac{\sum (a-b)^{2}}{\sum a}$

 

                                  $\Leftrightarrow S_{a}\left ( b-c \right )^{2}+S_{b}\left ( c-a \right )^{2}+S_{c}\left ( a-b \right )^{2}\geq 0$ (luôn đúng ) 

   

      Tại :

                                       $S_{a}=\frac{1}{c}-\frac{1}{a+b+c}=\frac{a+b}{c\left ( a+b+c \right )}>0$ , tương tự với $S_{b},S_{c}>0

              Dễ thấy :

 

                                                $\sum a^{2}\geq \frac{(\sum a)^{2}}{3}=\sum a\Rightarrow \frac{3\sum a^{2}}{\sum a}\geq 3$

 

                      Từ đây ta có điều phải chứng minh                                                                

Câu này bạn giải sai ngay từ bước đầu tiên :(


Ngài có thể trói cơ thể tôi, buộc tay tôi, điều khiển hành động của tôi: ngài mạnh nhất, và xã hội cho ngài thêm quyền lực; nhưng với ý chí của tôi, thưa ngài, ngài không thể làm gì được.

#5
hoanglong2k

hoanglong2k

    Trung úy

  • Điều hành viên THCS
  • 965 Bài viết

 Câu 7 có lẽ dễ nhất :)

 Áp dụng BĐT Cauchy-Schwarz ta có :

 $\sum \sqrt{a^2+abc}=\sum \sqrt{a}.\sqrt{a+bc}\leq \sqrt{(a+b+c)(a+b+c+ab+bc+ca)}\leq \frac{2}{\sqrt3}$

 Bộ anh tính bỏ thiệt à :( Sẽ cầm kiếm lên vào một ngày không xa chứ :(



#6
Senju Hashirama

Senju Hashirama

    Hạ sĩ

  • Thành viên
  • 69 Bài viết

Câu này bạn giải sai ngay từ bước đầu tiên :(

bạn nêu rõ chỗ sai của mình được chứ  :(  :(



#7
Hoang Nhat Tuan

Hoang Nhat Tuan

    Hỏa Long

  • Thành viên
  • 974 Bài viết

Ban đầu bạn chứng minh:$15-4\sum ab\geq 3$ phải không

Sau đó bạn lại đi chứng minh:

$\sum \frac{a^2}{b}\geq \frac{3\sum a^2}{\sum a}\geq 3$

Từ đó suy ra điều phải chứng minh.

Như thế là không đúng, vẫn chưa thể khẳng định được

Ví dụ có 3 số a,b,c, nếu $a\geq c$ và $b\geq c$ thì không thể khẳng định $a\geq b$ ngay được :D


Ngài có thể trói cơ thể tôi, buộc tay tôi, điều khiển hành động của tôi: ngài mạnh nhất, và xã hội cho ngài thêm quyền lực; nhưng với ý chí của tôi, thưa ngài, ngài không thể làm gì được.




1 người đang xem chủ đề

0 thành viên, 1 khách, 0 thành viên ẩn danh